Search found 101 matches


For example, if dimensions are 1*2200 then the road roller will need less rotations to cover the 1 m and more rotations to cover the 2200 m. If the dimensions are 11*200 then the road roller will require more rotations than previous example while covering 11 m( 11 against 1) and lesser rotations as ...

by everything's eventual

Wed Sep 19, 2012 7:21 pm
Forum: Data Sufficiency
Topic: DS - 2
Replies: 7
Views: 1538

For any of the dimensions that you have mentioned, the area of the ground remains the same. Here we need to find out how much area the road roller covers during each rotation and then divide 2200 by that area to get total number of rotations.. This can be done by using both given statements.

by everything's eventual

Wed Sep 19, 2012 7:18 pm
Forum: Data Sufficiency
Topic: DS - 2
Replies: 7
Views: 1538

Hello gmatdriller..can you provide the OA please... it help us understand whether we are correct or wrong and in turn helps us improve...

Thank You.

by everything's eventual

Wed Sep 19, 2012 7:13 pm
Forum: Critical Reasoning
Topic: Eating warm food can prevent seasonal flu
Replies: 9
Views: 1921

IMO it is D

This option states that both groups have an equal chance to be convicted, so expensive lawyers are the reason for a lower conviction rate among the group of financial criminals.

by everything's eventual

Wed Sep 19, 2012 6:09 pm
Forum: Critical Reasoning
Topic: Private defense vs public
Replies: 6
Views: 1756

This is my take : The argument is that Company X will be able to recoup the losses by reducing its workforce and therefore save a great deal in payroll expenditures. [spoiler]A)[/spoiler] The argument just says that the company will save a great deal in payroll expenditure. The argument does not dep...

by everything's eventual

Wed Sep 19, 2012 5:59 pm
Forum: Critical Reasoning
Topic: CR problem
Replies: 10
Views: 2229
by everything's eventual

Wed Sep 19, 2012 1:25 am
Forum: Data Sufficiency
Topic: Descriptive Statistics
Replies: 5
Views: 1999

Dear GMAT GURUNY


The integers are distinct and between 0 and 9, inclusive.
I am sorry but I do not understand why we should not consider negative numbers too.

by everything's eventual

Wed Sep 19, 2012 1:23 am
Forum: Data Sufficiency
Topic: Descriptive Statistics
Replies: 5
Views: 1999

The sum of the 5 distinct integers is 25. The sum of the two number which were removed is 13. Sum of remaining 3 numbers is 12. 1) Exactly 3 of the integers are consecutive primes. Lets consider 3,5 and 7. Sum of these 3 are 15. Remaining is 10. We can have 2 + 8 = 10 or 1 + 9 = 10 or even -2 + 12. ...

by everything's eventual

Wed Sep 19, 2012 1:21 am
Forum: Data Sufficiency
Topic: Descriptive Statistics
Replies: 5
Views: 1999

Is 2X - 3Y < X^2. ( Am I right in assuming it is x^2?) 1) 2X - 3Y = -2 This statement says that 2x-3y is negative. x^2 will always be positive. Therefore, 2x - 3y < x^2. Sufficient. 2) X >2 and Y>0 Let x = 3 and y = 1 2(3) - 3 (1) = 3 x^2 = 3*3 = 9 3 is less than 9 Let x = 3 and y = 10 2(3) - 3(10) ...

by everything's eventual

Wed Sep 19, 2012 12:39 am
Forum: Data Sufficiency
Topic: DS
Replies: 2
Views: 1391

How did you negate E) ? Take the smallest +ve even integer viz. 2

2 *3 *4 is divisible by 4. Take n as any even +ve integer and for that value n(n+1)(n+2) will always be divisible by 4.

eg : 4*5*6 , 24*25*26 , 34*35*36 just to name a few of many.

Regards.

by everything's eventual

Wed Sep 19, 2012 12:32 am
Forum: Problem Solving
Topic: even/odd integer
Replies: 5
Views: 1505

Hello All, my take : B) : If Gally offers contracts to refineries with extremely active maintenance departments then there is a good chance that Gally will not win those contracts. If these refineries already have an active maintenance department then they do not need a third party to do the same wo...

by everything's eventual

Mon Sep 17, 2012 6:39 pm
Forum: Critical Reasoning
Topic: Gally Pumps starting a new business
Replies: 4
Views: 1442

Hello Anil, if k = 1, then k-1 = 1-k so IMO we cannot def conclude that @ = '-'.

IMO answer is [spoiler]B)[/spoiler]

by everything's eventual

Mon Sep 17, 2012 5:56 pm
Forum: Data Sufficiency
Topic: OG 13, operator problem
Replies: 4
Views: 1371

everything's eventual wrote: Statement 2) says c = 36 = 3^2 * 2^2 Equating both sides we get (5^4)^b = 2 Then we get b = 0 and 2) should be sufficient. I am not sure how you are getting b=0 ?? Can you please elaborate !! Hello, I messed up the whole question. Sorry. I had a brain freeze. First of a...

by everything's eventual

Mon Sep 17, 2012 5:44 pm
Forum: Data Sufficiency
Topic: The value of B ?
Replies: 7
Views: 1414

hmmm..I am not sure how you did it...but this was my method :

Statement 1) is def not sufficient.

Statement 2) says c = 36 = 3^2 * 2^2

Equating both sides we get (5^4)^b = 2

Then we get b = 0 and 2) should be sufficient.

by everything's eventual

Mon Sep 17, 2012 2:25 am
Forum: Data Sufficiency
Topic: The value of B ?
Replies: 7
Views: 1414

It is not explicitly mentioned in the passage that Gary has not taken a single sick day from work over the last five years. Warm food can prevent flu is already mentioned in the passage and does nothing to strengthen the passage. Having said the above, if this were a GMAT question I would not choose...

by everything's eventual

Mon Sep 17, 2012 2:19 am
Forum: Critical Reasoning
Topic: Eating warm food can prevent seasonal flu
Replies: 9
Views: 1921